\section{Defining $e$} \problem{} Recall and prove the binomial theorem. \begin{solution} The binomial theorem: $$(a + b)^n = \sum_{i=0}^{n} \binom{n}{i}a^ib^{n-i}$$ We usually prove this by induction. \end{solution} \vfill \problem{} Prove the following: $$ \bigg(1 + \frac{1}{n} \bigg)^n < 3 - \frac{1}{n}\ \text{for all } n = 3, 4, 5, ...$$ This proves that that the sequence $e_n = (1 + \frac{1}{n})^n$, $n = 1, 2, ...$ is bounded from above, \par since $e_n < 3\ \forall n \in \mathbb{N}$. \begin{solution} $$ \bigg(1 + \frac{1}{n}\bigg)^n = \sum_{k=0}^n \binom{n}{k}\frac{1}{n^i} = 2 + \sum_{k=2}^n \binom{n}{k}\frac{1}{n^i} = 2 + \sum_{k=2}^n \frac{1}{k!}\frac{(n)(n-1)...(n-k+1)}{n^k} < 2 + \sum_{k=2}^n \frac{1}{k!} $$ $$ 2 + \sum_{k=2}^n \frac{1}{k!} < 2 + \sum_{k=2}^n \frac{1}{k(k-1)} = 2 + 1 - \frac{1}{n} = 3 - \frac{1}{n} $$ \end{solution} \vfill \pagebreak \theorem{Bernoulli's inequality} $(x + 1)^n \geq 1 + nx$ for $x \geq -1$ and $n \in \mathbb{N}$ \problem{} Use induction to prove \ref{bernoulli} \vfill \problem{} Use \ref{bernoulli} to prove that the $e_n$ defined in \ref{e_n} is monotonically increasing. \begin{solution} We want to show that the following is true: $$ \bigg( 1 + \frac{1}{n + 1} \bigg)^{n+1} > \bigg( 1 + \frac{1}{n} \bigg)^n $$ This inequality is equivalent to $$ \Bigg( \frac{ 1 + \frac{1}{n+1} }{ 1 + \frac{1}{n} } \Bigg)^{n+1} > \bigg( 1 + \frac{1}{n} \bigg)^{-1} = \frac{1}{1 + \frac{1}{n}} = \frac{n}{n+1} = 1 - \frac{1}{n+1} $$ Also, $$ \frac{ 1 + \frac{1}{n+1} } { 1 + \frac{1}{n} } = 1 - \frac{1}{(n + 1)^2} $$ \ref{bernoulli} tells us that $$ \bigg( 1 - \frac{1}{(n+1)^2} \bigg) ^ {n+1} = 1 - \frac{n+1}{(n+1)^2} = 1 - \frac{1}{n+1} $$ Since this is equivalent to our original inequality, we are done. \end{solution} \vfill \pagebreak \definition{} \ref{e_n}, \ref{e_n_inc}, and \ref{limexists} tell us that $e_n$ has a limit. \par Let us define $e$: $$e = \lim_{n\to\infty}{e_n} = \lim_{n\to\infty}{\bigg(1 + \frac{1}{n} \bigg)^n}$$ \vfill \problem{} Derive the formula for $V(t)$ if the annual rate $r$ is compounded continuously. \begin{solution} For a rate $r$ compounded $n$ times per year, we have $V(t) = P(1 + \frac{r}{n})^{nt}$. $\lim_{n\to\infty}{(V(t))} = \lim_{n\to\infty}{P(1 + \frac{r}{n})^{nt}}$ Let $a = \frac{n}{r}$ \par $\lim_{n\to\infty}{a} = \infty$, so $\lim_{n\to\infty}{V(t)} = \lim_{a\to\infty}{V(t)}$ \par Substituting $a$ for $n$, we get \par $P(1 + \frac{r}{n})^{nt} = P(1 + \frac{1}{a})^{art}$ \par And finally, we can evaluate \par $\lim_{a\to\infty}{P (1 + \frac{1}{a})^{art}} = Pe^{rt}$ \end{solution} \vfill \pagebreak